Saturday, February 27, 2010

Heart Murmur Case 2

A 65-year-old man presents to his primary care physician complaining of dyspnea, chest pain, and several syncopal episodes. His symptoms have worsened over the past few months and his third syncopal episode prompted this visit. On examination, a systolic ejection murmur is
auscultated with an ejection click in the right second intercostal space. Rales are present at the lung bases. He has a history of rheumatic fever in his twenties.

Q 1
Which of the following is the most likely diagnosis?
/ A. Aortic regurgitation
/ B. Aortic stenosis
/ C. Mitral stenosis
/ D. Tension pneumothorax
/ E. Thoracic aortic dissection

Q 2
Patients with syncope cannot maintain sufficient cardiac output to meet peripheral perfusion demands. Which of the following best describes cardiac output?
/ A. Cardiac output = end diastolic volume - end systolic volume
/ B. Cardiac output = heart rate X mean arterial pressure
/ C. Cardiac output = heart rate X stroke volume
/ D. Cardiac output = stroke volume X mean arterial pressure
/ E. Cardiac output = systemic vascular resistance X mean arterial pressure

Q 3
Which of the following might explain the angina pectoris in this patient
/ A. Increased ventricular wall tension limits perfusion
/ B. Left ventricular hypertrophy accelerates atherosclerotic diseas
/ C. Pulmonary hypertension decreases the PO2 of arterial blood
/ D. Stenotic valves occlude the coronary arteries
/ E. Twisting of the heart on its axis limits coronary flow

Q 4
Which of the following sets of changes depict the mean arterial pressure (MAP), Ieft ventricular peak systolic pressure (LVPSP), pulmonary
wedge pressure (PWP), and left atrial pressure (LAP) in this patient, compared to a healthy individuaI?

____________________________________________________________________

Heart Murmur Case 2 Answers

A1
The correct answer is B. Exertional dyspnea, angina pectoris, and syncope are the cardinal symptoms of aortic stenosis. Exam findings of systolic ejection murmur with ejection click confirm this diagnosis. Patients experience dyspnea because of the pulmonary edema generated by increased pulmonary capillary pressure, transmitted from increased left heart pressures. Angina pectoris results from left ventricular hypertrophy and increased interventricular pressures. Thus, an increased myocardial mass increases myocardial oxygen demand, while the increased wall pressures decrease perfusion. Ischemia results. Syncope results when the impeded left ventricle cannot meet peripheral perfusion demands.
Patients would have a diastolic murmur with aortic regurgitation (choice A).
Patients will have fatigue and dyspnea with mitral stenosis (choice C), but they have a diastolic murmur.
Patients with tension pneumothorax (choice D) present emergently with acute onset of shortness of breath and hemodynamic instability.
Thoracic aortic dissection (choice E) presents as acute onset of "tearing" chest pain that radiates to the back. Murmur may also be present.

A2
The correct answer is C. Cardiac output = heart rate X stroke volume.
Choice A is incorrect: Stroke volume = end diastolic volume - end systolic volume
Choice B is incorrect: Double product (estimation of cardiac work) = mean arterial pressure X heart rate
Choices D and E are nonsense distracters.

A3
The correct answer is A. The angina pectoris seen in aortic stenosis is caused by left ventricular hypertrophy. The ventricle must generate greater pressures to overcome the occluded outflow tract, and hypertrophy occurs. This contributes to cardiac ischemia in several ways. As noted above, an increased myocardial mass increases myocardial oxygen demand, while the increased wall tension decreases perfusion. The myocardium is perfused during diastole, and coronary perfusion relies on this relaxation. When mural diastolic pressures remain elevated in hypertrophy, perfusion is limited and ischemia results.
Hypertrophy does not accelerate atherosclerosis (choice B).
Pulmonary hypertension (choice C), seen in aortic stenosis, results in pulmonary edema and a thus a mild hindrance to alveolar gas exchange. One would not expect the PO2 to decrease enough to cause angina.
Stenotic valves do not occlude the coronary arteries (choice D).
The heart does not twist to occlude the coronary arteries (choice E).

A4
The correct answer is A. In aortic stenosis, the blood is ejected from the left ventricle through a smaller than normal opening. Because the resistance to ejection of blood is high, the left ventricular peak systolic pressure can sometimes increase to over 250 mm Hg with normal pressures in the aorta. The increase in left ventricular pressure raises left atrial pressure as well as pulmonary wedge pressure (which is a clinical index of left atrial pressure).

Heart Murmur Case 1

A pediatrician sees a 4-month-old boy for the first time. He had been delivered at home by his maternal grandmother, who had been a midwife in Southeast Asia before the family immigrated to the United States. According to the mother, the baby had been born on time and had
weighed about 7 pounds at birth. Physical examination is remarkable for a continuous murmur heard best at the upper left sternal border. A thrilI, analogous to a kitten's purring, can be felt over the left side of the baby's chest.

Q 1

The infant's murmur is suggestive of which of the following diagnoses?
/ A. Coarctation of the aorta
/ B. Hypoplastic left ventricle
/ C. Mitral valve stenosis
/ D. Patent ductus arteriosus
/ E. Tricuspid valve stenosis

Q 2
The infant's pulse at the wrist is noted to be full and to have a widened pulse pressure. On chest x-ray, the left atrium, Ieft ventricle, and ascending aorta are enlarged, and there is increased pulmonary blood flow. This infant's murmur is most likely related to which of the
following?
/ A. BIood flowing across the aortic valve
/ B. BIood flowing from the aorta to the pulmonary artery
/ C. BIood flowing from the left ventricle to the right ventricle
/ D. BIood flowing from the pulmonary artery to the aorta
/ E. BIood flowing from the right ventricle to the left ventricle

Q 3
Which of the following sets of changes depict the blood oxygen tensions at various locations in this patient compared to those of a healthy
infant?


Q 4
This baby's lesion accounts for approximately what percentage of congenital heart defects?
/ A. 1%
/ B. 10%
/ C. 25%
/ D. 50%
/ E. 95%

Q 5
If this baby's problem had been identified in his first week of life, which of the following medications might have been used to try to medically
correct his problem?
/ A. Acetaminophen
/ B. Codeine
/ C. Hydrocortisone
/ D. Indomethacin
/ E. Morphine

Q 6
Untreated patients with this baby's condition have been associated with which of the following mortality rates by 20 years of age?
/ A. 5%
/ B. 20%
/ C. 50%
/ D. 80%
/ E. 95%

____________________________________________________________________
Heart Murmur Case 1 Answers

A1
The correct answer is D. This is the murmur that is characteristic of patent ductus arteriosus.
Coarctation of the aorta (choice A) may produce a soft bruit heard over the coarctation site, often heard best in the back.
Hypoplastic left ventricle (choice B) per se does not usually produce a murmur, although a compensating patent ductus arteriosus or septal defect may produce a murmur.
Mitral valve stenosis (choice C) can produce an rumbling apical diastolic murmur.
Tricuspid valve stenosis (choice E) can produce a presystolic murmur heard at the left sternal edge in the 4th intercostal space.

A2
The correct answer is B. The ductus arteriosus is an important prenatal vessel that connects the aorta to the pulmonary artery, and allows blood in the (non-breathing) fetus to bypass the lungs. This vessel normally closes within a few hours to days of birth. In a baby without other congenital cardiac malformations, the blood flowing through a patent ductus arteriosus flows from the aorta to the pulmonary artery. In babies with other congenital cardiac malformations, the blood may flow in either direction (including choice D), depending on the specific malformation present. In this case, the widened pulse pressure, the enlarged chambers on the left side of the heart, and the increased pulmonary blood flow all suggest that the blood is flowing from the systemic circulation to the pulmonary system.
Blood flow across the aortic valve (choice A) per se would not cause increased pulmonary blood flow.
Blood flowing through a ventricular septal defect (choices C and E) would tend to produce enlargement of both ventricles without ascending aorta enlargement.

A3

The correct answer is A. Oxygen-rich blood from the aorta flows through the patent ductus arteriosus, increasing the oxygen tension in the pulmonary artery. Blood becomes fully oxygenated in the lungs in a normal way so that the oxygen tension of blood in the left ventricle, aorta, and vena cava is normal.

A4
The correct answer is B. Patent ductus arteriosus accounts for approximately 10% of all congenital heart defects. Many of the affected babies are premature. Other associations include birth asphyxia, rubella, coarctation of the aorta, ventricular septal defect, and trisomies 18 and 21.

A5
The correct answer is D. Before closing a patent ductus arteriosus, it is important to establish that the ductus is not compensating for other cardiac malformations, since a patent ductus arteriosus is beneficial in some other congenital lesions, including tetralogy of Fallot, Eisenmenger syndrome due to right ventricular hypertrophy, a large interventricular septal defect, or an interrupted aortic arch (in which the patent ductus supplies blood to the distal aorta). Indomethacin is the drug most commonly used to treat a patent ductus arteriosus, and it is usually used in the first few weeks of life. An alternative drug sometimes used is ibuprofen. These drugs act by inhibiting by decreasing the activity of cyclooxygenase and thereby inhibiting the production of prostaglandins. The drugs cause the ductus to narrow, and may have decreasing urine output as a side effect. If the baby is at risk of pulmonary edema, diuretics and/or modest fluid restriction may also be added. The indomethacin course may need to be repeated if it is not initially effective. In babies in whom medical treatment was ineffective or in whom the diagnosis was made late, surgical correction by either ligation of the vessel or implanting of a coil within the vessel is often used.
Acetaminophen (choice A), codeine (choice B), and morphine (choice E) are analgesics used for pain control that do not have any effect on the ductus arteriosus.
Hydrocortisone (choice C) is a corticosteroid and would not be used to close a patent ductus.

A6
The correct answer is B. While patients with very small amounts of blood flow through a patent ductus arteriosus may do fine throughout life, you should be aware that this is not a completely benign condition. Complications of patent ductus arteriosus include left heart failure, pulmonary hypertension, right heart hypertrophy and failure, bacterial endocarditis, myocardial ischemia, and necrotizing enterocolitis. It is estimated that the untreated mortality rate by age 20 years is 20%. Additionally, there is an estimated untreated mortality rate of 42% by age 45 and 60% by age 60.